GMAT Official Guide 2019 The research funds of a certain

This topic has expert replies
Moderator
Posts: 7187
Joined: Thu Sep 07, 2017 4:43 pm
Followed by:23 members

Timer

00:00

Your Answer

A

B

C

D

E

Global Stats

The research funds of a certain company were divided among three departments, X, Y, and Z. Which one of the three departments received the greatest proportion of the research funds?

(1) The research funds received by departments X and Y were in the ratio 3 to 5, respectively.
(2) The research funds received by departments X and Z were in the ratio 2 to 1, respectively.

GMAT/MBA Expert

User avatar
Elite Legendary Member
Posts: 10392
Joined: Sun Jun 23, 2013 6:38 pm
Location: Palo Alto, CA
Thanked: 2867 times
Followed by:511 members
GMAT Score:800

by [email protected] » Tue Jul 03, 2018 9:52 am

Timer

00:00

Your Answer

A

B

C

D

E

Global Stats

Hi All,

We're told that the research funds of a certain company were divided among three departments, X, Y, and Z. We're asked which one of the three departments received the greatest proportion of the research funds. This question can be solved in a number of different ways, including by TESTing VALUES.

1) The research funds received by departments X and Y were in the ratio 3 to 5, respectively.

Fact 1 gives us a ratio of X to Y (X:Y = 3:5), so we know that Y received a larger proportion of the funds than X did. But we don't know how much Z received.
IF...
X=3, Y=5, Z=1, then the answer to the question is Department Y.
X=3, Y=5, Z=10, then the answer to the question is Department Z.
Fact 1 is INSUFFICIENT

2) The research funds received by departments X and Z were in the ratio 2 to 1, respectively.

Fact 2 gives us a ratio of X to Z (X:Z = 2:1), so we know that X received a larger proportion of the funds than Z did. But we don't know how much Y received.
IF...
X=2, Y=1, Z=1, then the answer to the question is Department X.
X=2, Y=10, Z=1, then the answer to the question is Department Y.
Fact 2 is INSUFFICIENT

Combined, we have the following ratios:
X:Y = 3:5
X:Z = 2:1
We can 'combine' those ratios (multiply the first ratio by 2 and the second ratio by 3), which gives us:
X:Y = 6:10
X:Z = 6:3
X:Y:Z = 6:10:3
Thus, Department Y will ALWAYS get the most funding.

Final Answer: C

GMAT assassins aren't born, they're made,
Rich
Contact Rich at [email protected]
Image

GMAT/MBA Expert

User avatar
GMAT Instructor
Posts: 7247
Joined: Sat Apr 25, 2015 10:56 am
Location: Los Angeles, CA
Thanked: 43 times
Followed by:29 members

Timer

00:00

Your Answer

A

B

C

D

E

Global Stats

BTGmoderatorDC wrote:
Sun Jul 01, 2018 6:49 pm
The research funds of a certain company were divided among three departments, X, Y, and Z. Which one of the three departments received the greatest proportion of the research funds?

(1) The research funds received by departments X and Y were in the ratio 3 to 5, respectively.
(2) The research funds received by departments X and Z were in the ratio 2 to 1, respectively.
Solution:


Question Stem Analysis:

We need to determine which department (X, Y, or Z) received the greatest proportion of research funds.

Statement One Alone:

We see that the ratio of research funds received by departments X and Y were in the ratio 3 to 5. In other words, department Y received a greater proportion of funds than department X. However, since we don't have any information about funds received by department Z, statement one alone is not sufficient.

Statement Two Alone:

We see that the ratio of research funds received by departments X and Z were in the ratio 2 to 1. In other words, department X received a greater proportion of funds than department Z. However, since we don't have any information about funds received by department Y, statement two alone is not sufficient.

Statements One and Two Together:

With the two statements, we see that department X received a greater proportion of funds than department Z and department Y received a greater proportion of funds than department X. Thus, department Y must have received the greatest proportion of funds. The two statements together are sufficient.

Answer: C

Scott Woodbury-Stewart
Founder and CEO
[email protected]

Image

See why Target Test Prep is rated 5 out of 5 stars on BEAT the GMAT. Read our reviews

ImageImage